Q=1/6p^2


p= 13. 6 correct to 3 significant figures.


By considering bounds, work out the value of q to a suitable degree of accuracy.


Give a reason for your answer.


+

Answers

Answer 1

The value of Q, taking into account the significant figures is 30.8.

To work out the value of Q given the value of p, we can substitute the value of p into the equation Q = (1/6) × p².

Given p = 13.6, we can calculate Q as follows:

Q = (1/6) × (13.6)²

Q = (1/6) × 184.96

Q = 30.826666...

Now, let's consider the significant figures of the given value of p, which is 13.6 (3 significant figures).

Since the value of p has 3 significant figures, we should round our final answer for Q to 3 significant figures as well.

Considering the value of Q to a suitable degree of accuracy, we can round our answer to three significant figures, which gives us:

Q = 30.8

Therefore, the value of Q, taking into account the significant figures, is  30.8.

To learn more on Number system click:

https://brainly.com/question/22046046

#SPJ12


Related Questions

Ken filled out this information on the back of his bank statement. find ken’s revised statement balance. does his account reconcile?

Answers

If you have access to the information on the back of Ken's bank statement, you can calculate his revised statement balance by adding any credits and subtracting any debits from the previous statement balance.

Find out Ken revised statement balance?

Reconciling a bank account involves comparing the transactions in your own financial records with those listed on your bank statement. The goal is to ensure that the account balance in your financial records matches the balance reported by the bank.

To reconcile a bank account, you typically start with the ending balance on the previous bank statement, which becomes the beginning balance on the current statement. You then compare this balance with the transactions listed on the current statement, adding any credits (such as deposits or interest payments) and subtracting any debits (such as withdrawals or fees).

The resulting balance should match the ending balance listed on the current bank statement. If it does not match, then there may be errors or discrepancies in the account that need to be investigated. This can involve reviewing bank records, receipts, and other financial documents to identify any errors or missing transactions.

Reconciling your bank account on a regular basis is important for ensuring the accuracy of your financial records and identifying any issues or errors in a timely manner. It can also help you identify areas where you may be overspending or where you can save money by reducing fees or optimizing your financial habits.

Whether or not Ken's account reconciles depends on whether the calculated revised statement balance matches the bank's reported statement balance. If they match, then the account is reconciled. If they do not match, then there may be discrepancies in the account that need to be investigated and resolved.

Learn more about Bank

brainly.com/question/24014077

#SPJ11

A new sign is being designed for the cityâs skate park. Knowing the exact angles is necessary for fitting the sign where it will hang. The architect started to write in the angles, but went home sick before she could finish. It is up to you to fill in the missing angles. For 4 of the 8 missing angles, explain your answer

Answers

Using trigonometry, we can solve for the missing angles to find them as 18.43°, 45°, 45°, and 18.43°.

The sign is mounted on a sloped surface, which means that we'll need to use some trigonometry to find the missing angles.

Let's concentrate on the sign's upper right corner, where the letters x and y are absent from two perspectives. The magnitude of angle x may be determined using trigonometry.

Let's begin by sketching a right triangle that has an angle x. The triangle's two sides may be represented by the sign's vertical and horizontal lines, with the addition of a third side to join the top right corner of the sign to the sloping area below.

Since the sign is an octagon, we know that each interior angle is 135°. Therefore, the measure of angle y must be:

y = 180 - 135 = 45°

Now, let's look at the right triangle that includes angle x. We know that the hypotenuse of the triangle is the sloped surface of the sign, which has a length of 4.5 meters. We also know that the opposite side of the triangle is the height of the sign above the ground, which has a length of 1.5 meters.

Using trigonometry, we can find the measure of angle x by taking the inverse tangent of the opposite side over the adjacent side:

tan(x) = opposite/adjacent = 1.5/4.5 = 1/3

x = tan⁻¹(1/3) ≈ 18.43°

Therefore, the measure of angle x is approximately 18.43 degrees.

Hence, using trigonometry, we can solve for the missing angles to find them as 18.43°, 45°, 45°, and 18.43°.

To learn more about trigonometry, refer to:

https://brainly.in/question/2685053

#SPJ4

After a windstorm, a leaning pole makes a 75° angle with the road surface. the pole casts a 15-foot shadow when the sun is at a 45° angle of elevation. about how long is the pole?

Answers

The pole is approximately 3.86 feet tall.

What is the length of a leaning pole that makes a 75° angle with the road surface, if it casts a 15-foot shadow when the sun is at a 45° angle of elevation?

Let's denote the height of the pole as "x" (in feet). From the problem, we know that the pole makes a 75° angle with the road surface, which means that the angle between the pole and the vertical is 90° - 75° = 15°.

Now, we can use the tangent function to find the height of the pole:

tan(15°) = x/15

Multiplying both sides by 15, we get:

x = 15 tan(15°) ≈ 3.86 feet

Learn more abut length

brainly.com/question/9842733

#SPJ11

An oil slick on a lake is surrounded by a floating circular containment boom. as the boom is pulled in, the circular containment area shrinks. if the radius of the area decreases at a constant rate of 7 m/min, at what rate is the containment area shrinking when the containment area has a diameter of 80m?

Answers

The containment area is shrinking at a rate of 280π m²/min when the diameter is 80m and the radius is decreasing at a constant rate of 7m/min.

What is the rate of containment area shrinkage?

Let's begin by first finding the radius of the containment area when its diameter is 80m.

The diameter of the containment area is 80m, so its radius is half of that:

[tex]r = 80m / 2 = 40m[/tex]

Now, we need to find the rate at which the containment area is shrinking when the radius is decreasing at a constant rate of 7m/min.

We can use the chain rule of differentiation to find this rate:

[tex]dA/dt = dA/dr * dr/dt[/tex]

where A is the area of the containment, t is time, r is the radius of the containment, and dA/dt and dr/dt are the rates of change of A and r with respect to time, respectively.

We know that dr/dt = -7 m/min (negative because the radius is decreasing), and we can find dA/dr by differentiating the formula for the area of a circle with respect to r:

A = π[tex]r^2[/tex]

[tex]dA/dr = 2πr[/tex]

So, when r = 40m, we have:

[tex]dA/dt = dA/dr * dr/dt[/tex]

= (2πr) * (-7)

= -280π [tex]m^2[/tex]/min

Therefore, the containment area is shrinking at a rate of 280π m^2/min when the radius is decreasing at a constant rate of 7m/min and the diameter of the containment area is 80m.

Learn more about Rate of Change.

brainly.com/question/29518179

#SPJ11

If (x,y) is the solution to the system of equations above, what is the value of x?

Answers

Answer:

x = 16

Step-by-step explanation:

Multiply the entire first equation by -5 and the entire second equation by 2.

You then get:

15x + 20y = 200

2x - 20y = 72

Add the two equations and you get:

17x = 272

Divide 17 from both sides and you get the answer you need:

x = 16

Camille brought $39.50 to the art supply store. She bought a brush, a sketchbook, and a paint set. The brush was
1
3
as much as the sketchbook, and the sketchbook cost
1
2
the cost of the paint set. Camille had $4.50 left over after buying these items.
What was the cost of each item?
Solve on paper. Then check your work on Zearn.

Answers

The cost of each item, obtained from the equation for the sum of the costs of the item are;

A brush costs $3.5

A sketchbook costs $10.5

A paint set costs $21

What is an equation?

An equation is a mathematical statement that expresses equivalence between two expression joined by an '=' sign.

The amount Camille brought to the art supply = $39.50

The cost of the brush = (1/3) × The cost of the sketchbook

Cost of the sketchbook = (1/2) × Cost of the paint set

Amunt Camille had left over = $4.50

The cost of the items Camille bought = $39.50 - $4.5 = $35

Let x represent the cost of the brush, let y represent the cost of the sketchbook and let z represent the cost of the paint set

Therefore, we get the following equation; x + y + z = 35

x = (1/3)·y

y = (1/2)·z

Which indicates;

x = (1/3) × (1/2)·z = (1/6)·z

From which we get; (1/6)·z + (1/2)·z + z = 35

(5/3)·z = 3

z = 35 × 3/5 = 21

The cost of a paint set, z = $21The cost of a brush, x = (1/6) × $21 = $3.5The cost of a sketchbook, y = (1/2) × $21 = $10.5

Learn more on writing equations here: https://brainly.com/question/18713037

#SPJ1

Find the necessary sample size.


A population is normal with a variance of 99. Suppose you wish to estimate the population mean μ. Find the sample size needed to assure with 68. 26 percent confidence that the sample mean will not differ from the population mean by more than 4 units.


A. 9


B. 7


C. 613


D. 25

Answers

To estimate a population mean with 68.26% confidence that the sample mean will not differ from the population mean by more than 4 units, a sample size of 7 is needed. So, the correct answer is B).

The formula to calculate the sample size needed to estimate the population mean with a specified margin of error, assuming the population standard deviation is known, is

n = ((z-score * σ) / E)²

where

n = sample size

z-score = the z-score corresponding to the desired confidence level (in this case, the 68.26% confidence level corresponds to a z-score of 1)

σ = population standard deviation

E = the desired margin of error

Substituting the given values, we get

n = ((1 * √(99)) / 4)²

n = 6.1875

Since we need to have a whole number for the sample size, we must round up to the nearest integer. Therefore, the necessary sample size is 7.

So, the answer is B) 7.

To know more about standard deviation:

https://brainly.com/question/23907081

#SPJ4

Please help and explain if possibile

Answers

The missing lengths of triangles are 5in, 5mi, 13.9km,13.3mi respectively.

What is triangle?

A triangle is a closed, two-dimensional geometric figure with three straight sides and three angles.

What is Pythagorean theorem?

The Pythagorean Theorem is a fundamental theorem in Euclidean geometry that relates to the three sides of a right-angled triangle.

According to given information:

Using the Pythagorean theorem [tex](a^2 + b^2 = c^2)[/tex], we can solve for the missing side in each triangle.

Triangle 1:

[tex]a = 12 in\\\\c = 13 in\\\\a^2 + b^2 = c^2\\\\12^2 + b^2 = 13^2\\\\144 + b^2 = 169\\\\b^2 = 25\\\\b = \sqrt{(25)}\\\\b = 5 in[/tex]

Therefore, the length of the missing side in Triangle 1 is 5 in.

Triangle 2:

[tex]a = 4 mi\\\\b = 3 mi\\\\c = x\\\\a^2 + b^2 = c^2\\\\4^2 + 3^2 = x^2\\\\16 + 9 = x^2\\\\25 = x^2\\\\x = \sqrt{(25)}\\\\x = 5 mi[/tex]

Therefore, the length of the hypotenuse in Triangle 2 is 5 mi.

Triangle 3:

[tex]a = x\\\\b = 11.9 km\\\\c = 14.7 km\\\\a^2 + b^2 = c^2\\\\x^2 + 11.9^2 = 14.7^2\\\\x^2 = 14.7^2 - 11.9^2\\\\x^2 = 192.36\\\\x = \sqrt{(192.36)}\\\\x = 13.9 km[/tex]

Therefore, the length of the height in Triangle 3 is 13.9 km.

Triangle 4:

[tex]a = x\\\\b = 6.3 mi\\\\c = 15.4 mi\\\\a^2 + b^2 = c^2\\\\x^2 + 6.3^2 = 15.4^2\\\\x^2 = 15.4^2 - 6.3^2\\\\x^2 = 178.09\\\\x = \sqrt{(178.09)}\\\\x = 13.3 mi[/tex]

Therefore, the length of the height in Triangle 4 is 13.3 mi.

To learn more about triangle visit:

https://brainly.com/question/1058720

#SPJ1

What is the height of the cylinder rounded to the nearest tenth? The figure * 1 point is not drawn to scale . V = 284.7 inches cubed

Answers

The height of the cylinder is 3.6 inches.

What is the height of the cylinder?

We know that the volume of a cylinder of radius R and height H is:

V = pi*R²*H

where pi = 3.14

We know that the radius is R = 5in and the volume is 284.7 inches cubed, replacing that in the formula above we will get:

284.7 in³= 3.14*(5 in)²*H

Solving that for H we will get:

H= (284.7 in³)/ 3.14*(5 in)²

H = 3.6 inches.

Learn more about cylinders at:

https://brainly.com/question/9554871

#SPJ1

What is the equation of the line that best fits the given data? A graph has points (negative 3, negative 3), (negative 2, negative 2), (1, 1. 5), (2, 2), (3, 3), (4, 4). A. Y = 2 x + 1 c. Y = x + 1 b. Y = x d. Y = negative x Please select the best answer from the choices provided A B C D Mark this and return

Answers

The equation of the line that best fits the given data is y = (5/6)x + 1/3

The equation of the line that best fits the given data can be found by using the slope-intercept form of a linear equation, which is y = mx + b, where m is the slope and b is the y-intercept. To find the slope, we can use the formula:

m = (y2 - y1) / (x2 - x1)

Using the points (1, 1.5) and (4, 4), we get:

m = (4 - 1.5) / (4 - 1) = 2.5 / 3 = 5/6

Now we can use one of the given points to find the y-intercept. Let's use the point (2, 2):

y = mx + b
2 = (5/6)(2) + b
2 = 5/3 + b
b = 2 - 5/3
b = 1/3

Therefore, the equation of the line that best fits the given data is:

y = (5/6)x + 1/3

The best answer is C. Y = x + 1.

To know more about equation refer here

https://brainly.com/question/29657983#

#SPJ11

An square aquarium which is 15cm long has 1250 millilitres of water how much more water needed to fill the aquarium completely

Answers

You need to add 2125 milliliters of water to fill the square aquarium completely.

We need to find the volume of the square aquarium and then determine the additional water needed to fill it completely. Here are the steps:

1. Convert the given length to meters: 15 cm = 0.15 m
2. Calculate the volume of the square aquarium: Volume = length × width × height. Since it's a square aquarium, all sides are equal, so Volume = 0.15 m × 0.15 m × 0.15 m = 0.003375 cubic meters.
3. Convert the volume to milliliters: 0.003375 cubic meters × 1,000,000 mL/cubic meter = 3375 mL.
4. Calculate the additional water needed: Total volume - Current volume = 3375 mL - 1250 mL = 2125 mL.

You need to add 2125 milliliters of water to fill the square aquarium completely.

Learn more about volume,

https://brainly.com/question/27710307

#SPJ11

Use the method of logarithmic differentiation to find the derivative of x^{sin x} with respect to x. (Your final answer should be in terms of x.) Hint: Let( y = x^{sin x})and your goal is to find dy/dx

Answers

The derivative of y = x^(sin x) with respect to x is:

dy/dx = x^(sin x) * (cos x * ln(x) + sin x * (1/x)).

To find the derivative of y = x^(sin x) with respect to x using logarithmic differentiation, follow these steps:
1. Take the natural logarithm of both sides of the equation:

ln(y) = ln(x^(sin x))
2. Use the properties of logarithms to simplify:

ln(y) = sin x * ln(x)
3. Differentiate both sides with respect to x, using the chain rule and product rule:

(1/y) * dy/dx = cos x * ln(x) + sin x * (1/x)
4. Multiply both sides by y to solve for dy/dx:

dy/dx = y * (cos x * ln(x) + sin x * (1/x))
5. Substitute the original expression for y (y = x^(sin x)) back into the equation:

dy/dx = x^(sin x) * (cos x * ln(x) + sin x * (1/x))
So the derivative of y = x^(sin x) with respect to x is:

dy/dx = x^(sin x) * (cos x * ln(x) + sin x * (1/x)).

To learn more about logarithmic differentiation visit : https://brainly.com/question/31299592

#SPJ11

Question 1(Multiple Choice Worth 4 points)


A funnel is shaped like a cone and is 4. 5 inches high and has a diameter of 6 inches. What is the volume of the funnel? Use 3. 14 for pi. Round your answer to the nearest hundredth. 10. 60 in3

42. 39 in3

63. 61 in3

169. 64 in3

Answers

The volume of the funnel is approximately 42.39 in³. The correct answer is option 2.

To calculate the volume of the funnel, which is shaped like a cone, we need to use the formula for the volume of a cone: V = (1/3)πr²h.

Given:
Height (h) = 4.5 inches
Diameter = 6 inches
Radius (r) = Diameter / 2 = 6 / 2 = 3 inches
Pi (π) ≈ 3.14

Now, plug the values into the formula:

V = (1/3) × 3.14 × 3² × 4.5
V ≈ (1/3) × 3.14 × 9 × 4.5
V ≈ 3.14 × 3 × 4.5
V ≈ 42.39 in³

So, the volume of the funnel is approximately 42.39 in³. The correct answer is option 2.

To learn more about volume, refer below:

https://brainly.com/question/1578538

#SPJ11

Pythagorean theorem
I need help with my math

Answers

The height of the flagpole is 26.0 feet.

What is height?

Height is the vertical distance between two points.

To calculate the height of the flagpole, we use the formula below.

Formula:

h = √(l²-d²)............... Equation 1

Where:

h = Height of the flagpolel = Length of the wired = Distance of the wire from the ground to the base of the pole

From the question,

Given:

l = 300 feetd = 15 feet

Substitute these values into equation 1

d = √(30²-15²)d = √675d = 26.0 feet

Learn more about height here: https://brainly.com/question/12446886

#SPJ1

Part A: Sydney made $18. 50 selling lemonade, by the cup, at her yard sale. She sold each cup for $0. 50 and received a $3 tip from a neighbor. Write an equation to represent this situation. (4 points)



Part B: Daria made a profit of $21. 00 selling lemonade. She sold her lemonade for $0. 75 per cup, received a tip of $3 from a neighbor, but also had to buy each plastic cup she used for $0. 10 per cup. Write an equation to represent this situation. (4 points)



Part C: Explain how the equations from Part A and Part B differ. (2 points)

Answers

Part A: The equation to represent this situation is: 0.50x + 3 = 18.50
Part B: The equation to represent this situation is: 0.75x + 3 - 0.10x = 21.00
Part C: The equations differ in the following ways:
1. Sydney's equation involves only the price per cup and the tip, while Daria's equation also considers the cost of the plastic cups.
2. The price per cup for Sydney and Daria are different.

Part A: The equation to represent this situation is:

18.50 = 0.50x + 3

Where x represents the number of cups of lemonade sold.

Part B: The equation to represent this situation is:

21.00 = 0.75x + 3 - 0.10x

Where x represents the number of cups of lemonade sold.

Part C: The equations from Part A and Part B differ in that Part B takes into account the cost of each plastic cup used to serve the lemonade, while Part A only considers the revenue from selling each cup of lemonade and the tip received. This means that the profit in Part B is calculated after deducting the cost of each plastic cup from the revenue earned, while the profit in Part A does not account for any costs incurred.

To know more about equation, refer to the link below:

https://brainly.com/question/24870633#

#SPJ11

Find the derivative of y with respect to x if y= e⁻¹⁷ˣ.
dy/dx = ...

Answers

The derivative of y with respect to x when y = e⁻¹⁷ˣ  is dy/dx = -17eˣ

Find the derivative?

To find the derivative of y with respect to x when y = e⁻¹⁷ˣ, we'll use these terms: "derivative", "respect", and "with".

Identify the function. In this case, y = e⁻¹⁷ˣ

Find the derivative (dy/dx) of the function with respect to x. To do this, we'll apply the chain rule, which states that the derivative of a composite function is the derivative of the outer function times the derivative of the inner function.

The outer function is e^(u) (where u is the inner function), and its derivative with respect to u is e^(u). The inner function is -17x, and its derivative with respect to x is -17.

Apply the chain rule. The derivative of y with respect to x (dy/dx) is the product of the derivative of the outer function and the derivative of the inner function: (e^(u)) * (-17).

Substitute u with the inner function (-17x). So, dy/dx = (e⁻¹⁷ˣ * (-17).

The derivative of y with respect to x when y = e⁻¹⁷ˣ  is dy/dx = -17eˣ

Learn more about chain rule.

brainly.com/question/28972262

#SPJ11

47 students are picking two activities to do over the weekend.
7 picked painting and sport.
6 did not pick painting or sport.
Twice as many students picked sport than painting as one of their activities.
Find the amount that picked sport and not painting.

Answers

Let's use the following variables:
- P = number of students who picked painting
- S = number of students who picked sport
- N = total number of students (47)

From the problem, we know that:
- 7 students picked both painting and sport, so P ∩ S = 7
- 6 students did not pick painting or sport, so (N - P - S) = 6, which simplifies to N = P + S + 6
- Twice as many students picked sport than painting as one of their activities, so S = 2P

We want to find the number of students who picked sport but not painting, which is S - 7 (since 7 students picked both). Substituting S = 2P into the equations above, we get:

N = P + 2P + 6
N = 3P + 6
47 = 3P + 6
41 = 3P
P = 13.67 (rounded to the nearest hundredth)

Since P must be a whole number, we can round up to P = 14. Then, S = 2P = 28, and S - 7 = 21. Therefore, 21 students picked sport but not painting.

3 Let a represent a positive number and let b represent a negative number. Tell whether each statement is True or False. A. The difference (a - b) could be negative. True False True False b. The difference (b - a) cannot be positive. C. The sum (a + b) could be positive. True False d. The sum (b + a) must be negative. True False​

Answers

Using various laws of integers we can say that if a is a positive integer and b is a negative integer, statement A is False, B is True, C s True and D is false.

Here we are given that a is a positive integer while b is a negative integer.

A. The statement says that the difference (a - b) could be negative.

According to the subtraction law of integers, when a negative number is subtracted from a positive number, that is we have

2 - (-3)

Here the 2 minus signs will make a positive to give

2 + 3 = 5

Hence (a - b) will be a positive number since b is negative.

B.

The difference (b - a) cannot be positive.

Since a is positive and b is negative, according to the above example we will get

-3 - 2 = -5

Hence it is true that the difference (b - a) can't be positive.

C.

The sum (a + b) could be positive.

Here, we can see that a is a positive number while b is a negative number. In the light of above example, we will get

2 - 3 = -1

Here the sum is nagative as 3 > 2, but if we had

3 + (-2), then the answer would have been 1. Hence (a + b) can be positive. Hence the statement is true.

D.

The sum (b + a) must be negative.

Integers have commutative properties. Hence a + b = b + a

Hence if a + b can be positive, then b + a can also be positive.

Hence the statement is False.

To learn more about Integer visit

https://brainly.com/question/1768254

#SPJ4

In a survey, the planning value for the population proportion is p* = 0.25. How large a sample should be taken to provide a 95% confidence interval with a margin of error of 0.02? Round your answer up to the next whole number. How large a sample should be selected to provide a 95% confidence interval with a margin of error of 2? Assume that the population standard deviation is 30. Round your answer to next whole number.

Answers

The Sample size that is necessary for the selection is =7203

How to solve

Given that,

[tex]\hat p= 0.25[/tex]

[tex]1 - \hat p = 1 - 0.25 = 0.75[/tex]

margin of error = E = 0.01

At 95% confidence level the z is ,

\alpha = 1 - 95% = 1 - 0.95 = 0.05

[tex]\alpha / 2 = 0.05 / 2 = 0.025[/tex]

Z\alpha/2 = Z0.025 = 1.96 ( Using z table )

Sample size = n = [tex](Z\alpha/2 / E)2 * \hat p * (1 - \hat p)[/tex]

= (1.96 / 0.01)2 * 0.25 * 0.75

= 7203

Sample size =7203

In statistics, the sample size is the measure of the number of individual samples used in an experiment.

The size of the sample holds significant importance in any empirical study that aims to draw conclusions about a larger population based on a smaller sample.

Read more about sample size here:

https://brainly.com/question/30224379

#SPJ1

1 pts How much bubble wrap is needed to cover a cylindrical vase that is 16 inches tall with a diameter of 6 inches?​

Answers

415 square inches of bubble wrap to cover the cylindrical vase that is 16 inches tall with a diameter of 6 inches.

To calculate how much bubble wrap is needed to cover the cylindrical vase, you will need to find the circumference and height of the vase.

First, calculate the circumference of the vase using the diameter of 6 inches:
Circumference = π x diameter
Circumference = 3.14 x 6
Circumference = 18.84 inches

Next, calculate the height of the vase which is given as 16 inches.

To find the surface area of the vase, you will need to multiply the circumference by the height and add the area of the circular bases. The formula for the surface area of a cylinder is:

Surface area = 2πr² + 2πrh
where r is the radius and h is the height.

Since the vase has circular bases, we can find the area of each base by using the formula:
Area of circle = πr²

Now, let's find the radius of the vase:
[tex]Radius = \frac{diameter}{2}[/tex]
[tex]Radius = \frac{6}{2}[/tex]
Radius = 3 inches

So, the area of each base is:

Area of base = π x (radius)²
Area of base = π x 3²
Area of base = 28.27 square inches

The total area of the two bases is 2 x 28.27 = 56.54 square inches.

Now, let's find the surface area of the cylinder:

Surface area = 2πr² + 2πrh
Surface area = 2 x π x 3² + 2 x π x 3 x 16
Surface area = 113.1 + 301.44
Surface area = 414.54 square inches

Therefore, you would need approximately 415 square inches of bubble wrap to cover the cylindrical vase that is 16 inches tall with a diameter of 6 inches.

To know more about "surface area of a cylinder" refer here:

https://brainly.com/question/28575608#

#SPJ11

I need help. What would be the answer?

Answers

The answer would be DE/EC.

Answer:

Step-by-step explanation:

DE/EC.

20 pt if someone can answer this!!!
Pls help.

The median value is____

Answers

Answer:

The median value is 45.

Step-by-step explanation:

"The median is the middle number in a sorted, ascending or descending list of numbers"

The middle number here is 45

50

Step-by-step explanation:

I put the explanation on the attachment. please see it.

40 points!!!
Peyton's photo album has 6 1/2 pages of family photos and f pages of
photos of friends. Write an expression that shows the total number
of pages in Peyton's album. Then evaluate the expression if there are
3 1/2 pages of photos of friends.

Answers

The expression that shows the total number of pages in Peyton's album is  6 1/2 + f.

We are given that;

Number of pages= 6 1/2

Now,

To write an expression that shows the total number of pages in Peyton’s album, you need to add the number of pages of family photos and the number of pages of friends photos. The expression is:

6 1/2 + f

To evaluate the expression if there are 3 1/2 pages of photos of friends, you need to substitute f with 3 1/2 and then add the fractions. The answer is

6 1/2 + 3 1/2 = 10

Therefore, by the expression the answer will be 6 1/2 + f.

To know more about an expression follow;

brainly.com/question/19876186

#SPJ1

Please help ASAP!!!!! In a certain Spanish class of 30 students, 11 of them play basketball and 15 of them play baseball. There are 10 students who play both sports. What is the probability that a student chosen randomly from the class plays basketball or baseball? Answer


should be a fraction in simplest form

Answers

The probability that a student chosen randomly from the class plays basketball or baseball is 8/15

Total number of students in Spanish class = 30

Student who plays basketball (A) = 11

Student who plays baseball (B) = 15

Student who plays both sports (A and B) = 10

To find a student who plays basketball or baseball (A or B)

(A or B)  = A + B -  (A and B)

(A or B) = 11 +15 -10

(A or B) = 16

P(A or B) = No. of favorable outcome/ Total no. of outcomes

P(A or B) = 16/30

In simplest form = 8/15

To know more about probability click here :

https://brainly.com/question/30034780

#SPJ4

Help pls..
How many solutions does the system of linear equations represented in the graph have?

Coordinate plane with one line that passes through the points 0 comma negative 2 and 2 comma negative 1.

One solution at (−2, 0)
One solution at (0, −2)
Infinitely many solutions
No solution

Answers

The number of solutions which this system of linear equations represented in the graph have is: C. Infinitely many solutions.

How to determine an equation of this line?

In Mathematics and Geometry, the point-slope form of a straight line can be calculated by using the following mathematical expression:

y - y₁ = m(x - x₁)

Where:

x and y represent the data points.m represent the slope.

First of all, we would determine the slope of this line;

Slope (m) = (y₂ - y₁)/(x₂ - x₁)

Slope (m) = (-1 + 2)/(2 - 0)

Slope (m) = 1/2

At data point (0, -2) and a slope of 1/2, a linear equation for this line can be calculated by using the point-slope form as follows:

y - y₁ = m(x - x₁)

y + 2 = 1/2(x - 0)  

y = 1/2(x) - 2

Read more on point-slope here: brainly.com/question/24907633

#SPJ1

Let f(x) = 4x^3 – 3x^2 – 18x +5. (a) Find the critical numbers of f. (b) Find the open interval(s) on which f is increasing and the open interval(s) on which f is decreasing. (c) Find the local minimum value(s) and focal maximum value(s) of f, if any.
(d) Find the open interval(s) where f is concave upward and the open interval(s) where f is concave downward e) Find the inflection points of the graph of f, if any

Answers

(a) The critical numbers happen when x = 3 or x = -1/2

(b) f is decreasing on (-∞, -1/2), increasing on (-1/2, 3), and increasing on (3, ∞).

(c) f has a local minimum value of -22 at x = 3, and a local maximum value of 25.5 at x = -1/2.

(d) f is concave downward on (-∞, 1/4) and concave upward on (1/4, ∞).

(e) The inflection point of f is at x = 1/4.

(a) To find the critical numbers of f, we need to find the values of x where the derivative of f equals zero or does not exist.

f'(x) = 12x² - 6x - 18 = 6(2x² - x - 3) = 6(x - 3)(2x + 1)

Setting f'(x) equal to zero, we get:

6(x - 3)(2x + 1) = 0

x = 3 or x = -1/2

These are the critical numbers of f.

(b) To find the intervals where f is increasing and decreasing, we need to examine the sign of the derivative f'(x) in the intervals determined by the critical numbers.

When x < -1/2, f'(x) < 0, so f is decreasing on the interval (-∞, -1/2).

When -1/2 < x < 3, f'(x) > 0, so f is increasing on the interval (-1/2, 3).

When x > 3, f'(x) > 0, so f is increasing on the interval (3, ∞).

(c) To find the local minimum and maximum values of f, we need to examine the critical numbers and the end points of the intervals.

f(3) = 4(3)³ - 3(3)² - 18(3) + 5 = -22

f(-1/2) = 4(-1/2)³ - 3(-1/2)² - 18(-1/2) + 5 = 25.5

Thus, f has a local minimum value of -22 at x = 3, and a local maximum value of 25.5 at x = -1/2.

(d) To find the intervals where f is concave upward and concave downward, we need to examine the sign of the second derivative f''(x).

f''(x) = 24x - 6 = 6(4x - 1)

When x < 1/4, f''(x) < 0, so f is concave downward on the interval (-∞, 1/4).

1/4 < x, f''(x) > 0, so f is concave upward on the interval (1/4, ∞).

(e) To find the inflection points of f, we need to examine the points where the concavity changes.

The concavity changes at x = 1/4, which is the only inflection point o

Casho went shopping for a new pair of sneakers because of a sale. The price on the tag was $25, but Casho paid $22. 50 before tax. Find the percent discount

Answers

The percent discount on the sneakers is 10%

Casho paid $22.50 before tax, despite the item's $25 tag price. The discount is the difference between the original price and the sale price, which is $25 - $22.50 = $2.50.

The discount is the difference between the original price and the discounted price, expressed as a percentage of the original price.

To find the percent discount, we divide the discount by the original price and multiply by 100:

Percent discount = (discount / tag price) x 100

Percent discount = ($2.50 / $25) x 100

Percent discount = 0.1 x 100

Percent discount = 10%

Therefore, the percent discount on the sneakers is 10%

Learn more about Discount here

https://brainly.com/question/3541148

#SPJ4

Ricky has 23 hours each week to dedicate to his classes. homework takes 6.5 hours and each class (c) is 1.5 hours long. how many classes does ricky take? which equation models the question? explain your thinking.



a) 23=6.5-1.5c b) 23=6.5+1.5c



c) 23=1.5+6.5c d) 23=1.5-6.5c

Answers

by dividing both sides by 1.5.

How many classes does Ricky take?

To solve the problem, we need to first determine the total amount of time Ricky spends in his classes. We know that each class is 1.5 hours long, so if he takes c classes, then he will spend a total of 1.5c hours on class time. In addition, we know that he spends 6.5 hours on homework. Therefore, the total amount of time Ricky spends on his classes and homework is:

Total time = Class time + Homework time

Total time = 1.5c + 6.5

We also know that Ricky has 23 hours per week to dedicate to his classes and homework. Therefore, we can set up the following equation:

Total time = 23

Substituting the expression for a total time from the first equation, we get:

1.5c + 6.5 = 23

Now we can solve for c:

1.5c = 23 - 6.5

1.5c = 16.5

c = 11

Therefore, Ricky takes 11 classes.

The equation that models the question is b) 23=6.5+1.5c. This equation correctly represents the total time Ricky spends on his classes and homework (23 hours), as well as the time he spends on homework (6.5 hours) and the time he spends in class (1.5c hours).

by dividing both sides by 1.5.

Learn more about Ricky taking classes.

brainly.com/question/31598666

#SPJ11

The table shows the results of a survey of 150 students.


Use the table to find the probability of a student participating


in each sport.


1. Football


2. Tennis

Answers

Probability of a student participating in football: 0.4 or 40%

Probability of a student participating in tennis: 0.2 or 20%

Assuming that the table lists the number of students who participate in each sport out of a total of 150 students, we can find the probability of a student participating in each sport by dividing the number of students who participate in each sport by the total number of students:

Probability of a student participating in football:

Number of students who participate in football / Total number of students = P(Football)

Probability of a student participating in tennis:

Number of students who participate in tennis / Total number of students = P(Tennis)

For example, if the table shows that 60 students participate in football and 30 students participate in tennis out of a total of 150 students, then the probabilities would be:

Probability of a student participating in football:

60/150 = 0.4 or 40%

Probability of a student participating in tennis:

30/150 = 0.2 or 20%

To know more about probability, refer to the link below:

https://brainly.com/question/13515204#

#SPJ11

7. A rectangular prism has a volume of 135ft^3. The width of the rectangular prism is (2x+10)ft. The height of the rectangular prism is 5 times it's width. Write a expression that gives the length of the rectangular prism in feet?



A. 4(x+5)/27 B. 27/4(x+5)



C. (2x^2+100)/27. D. 27/(2x^2+100)

Answers

The expression that gives the length of the rectangular prism in feet is option D: 27/(2x^2+100).

What is the expression that gives the length of the rectangular prism in feet?

The volume of a rectangular prism is given by the formula V = lwh, where l is the length, w is the width, and h is the height.

We are given that the volume of the rectangular prism is 135ft^3, and the width is (2x+10)ft. Also, the height is 5 times the width, so h = 5w.

Substituting these values in the formula for the volume, we get:

135 = l(2x+10)(5w)

Dividing both sides by (2x+10)(5w), we get:

l = 135 / (2x+10)(5w)

l = 135 / [10(x+5)w]

Now we can substitute h = 5w:

l = 135 / [10(x+5)h/5]

l = 135 / [2(x+5)h]

l = 135 / [2(x+5)(5w)]

l = 135 / [10(x+5)^2]

Simplifying the expression, we get:

l = 27 / (2(x+5)^2)

Therefore, the expression that gives the length of the rectangular prism in feet is option D: 27/(2x^2+100).

Learn more about rectangular prism in feet

brainly.com/question/30956442

#SPJ11

Other Questions
(1 point) Calculate TT, and n(u, v) for the parametrized surface at the given point. Then find the equation of the tangent plane to the surface at that point. O(u, v) = (2u + 0.0 - 40, 8u); u= 3, U = According to the passage, for what aspect of a nuclear bomb attack did the government conclude there was no defense? Highly realistic figures and detailed scenes are characteristic of the ? Style of Greek pottery A 32" flat screen television measures 32 inches across its diagonal. the diagonal makes a 35 angle with the bottom of the television32"ha35select all equations that can be used to solve for the height, h, of the television screen A car travels at 54 km/h for first 20 s, 36 km/h for next 30 s and finally 18 km/h for next 10 s. Find its average speed. What is the surface of the prism Carl takes out a loan which accrues interest at a flat rate of 1.96% per quarter.what is the equivalent yearly simple interest rate? A and b verify that they possess a common key k, using a public one-wayfunction h, as follows:1. a sends h(h(k)) to b2. b verifies that the received value is correct3. b sends h(k) to a4. a verifies that the received value is correct The grouping of related activities into units is called______.a. coordinationb. division of laborc. unity of command and directiond. departmentalization mi hermano es el___ de mi padre. A. abuelo B.C.D.primo hijo padre Write an equation to show the total length of the bandages if they are placed end-to-endThere is an image attached btw Tamika wrote an integer. the opposite of tamika 's integer is -37. which of these statementsabout tamika's integer must be true?i. the integer is -37ii. the integer has an absolute value of -37iii. the integer is 37iv. the integer has an absolutevalue of 37a) i and iib) i and ivc) ii and iiid) iii and iv Novak Corp., opened an incorporated dental practice on January 1, 2022. During the first month of operations, the following transactions occurred.1. At January 31, $930 of such services was completed but not yet billed to the insurance companies.2. Utility expenses incurred but not paid prior to January 31 totaled $520. 3. Purchased dental equipment on January 1 for $87,600, paying $28,250 in cash and signing a $59,350, 3-year note payable interest is paid each December 31). The equipment depreciates $540 per month. Interest is $670 per month. 4. Purchased a 1-year malpractice insurance policy on January 1 for $24,000. 5. Purchased $2,320 of dental supplies (recorded as increase to Supplies). Prepare the adjusting entries on January 31. Account titles are Accumulated Depreciation Equipment, Depreciation Expense, Service Revenue, Accounts Receivable, Insurance Expense, Interest Expense, Interest Payable, Prepaid Insurance, Supplies, Supplies Expense, Utilities Expense, and Accounts Payable. In your own words, describe what a capella means. Find the terms: a=_1=8, r=0.7 2. A manufacturer of electronic kits has found that the mean time required for novices to assemble its new circuit tester is 3. 15 hours, with a sample standard deviation of 0. 23 hours. A consultant has developed a new instructional booklet intended to reduce the time an inexperienced kit builder will need to assemble the device. In a test of the effectiveness of the new booklet, 25 novices require a mean of 2. 98 hours to complete the job. Assuming the population of times is normally distributed, and using the 0. 01 level of significance, should we conclude that the new booklet is effective? Determine and interpret the p-value for the test ACTIVITY 2: Solve the following problems completely. Suppose you think Apple stock is going to appreciate substantially in value in the next year. Say the stocks current price, S0, is $120, and a call option expiring in one year has an exercise price, X, of $120 and is selling at a price, C, of $18. With $14,400 to invest, you are considering three alternatives. A. Invest all $14,400 in the stock, buying 120 shares. B. Invest all $14,400 in 800 options (8 contracts). C. Buy 100 options (one contract) for $1,800, and invest the remaining $12,600 in a money market fund paying 6% in interest over 6 months (12% per year) 10. explain one strength & one limitation of using models of internal city structure to predict internal city structure. What is the median?94 91 99 81 98 82 82